LSAT and Law School Admissions Forum

Get expert LSAT preparation and law school admissions advice from PowerScore Test Preparation.

 Brook Miscoski
PowerScore Staff
  • PowerScore Staff
  • Posts: 418
  • Joined: Sep 13, 2018
|
#65733
MrMola,

(C) is out of scope because it concerns making cars safer whereas the stimulus concerns changing how safety testing is done, not necessarily improving the safety effectiveness of the testing. If that's what you mean by "safety features," yes.
 theamazingrace
  • Posts: 59
  • Joined: Oct 17, 2020
|
#83769
Brook Miscoski wrote: Wed Jun 19, 2019 7:02 pm MrMola,

(C) is out of scope because it concerns making cars safer whereas the stimulus concerns changing how safety testing is done, not necessarily improving the safety effectiveness of the testing. If that's what you mean by "safety features," yes.
Would C also be wrong because it just repeats what is already said in the stimulus and doesn't add anything or rule out anything that would weaken the relationship presented? I choose C because I read the question too quickly and thought it was a most strongly supported question but after reading it again it is clearly a strength question so see now that new information is allowed.

Thanks
 Robert Carroll
PowerScore Staff
  • PowerScore Staff
  • Posts: 1787
  • Joined: Dec 06, 2013
|
#84058
Grace,

I agree with Brook that answer choice (C) is out of scope. It's not repeating the stimulus - the information in its necessary condition about producing safer cars is not relevant to the stimulus, which never mentions any such thing. I don't think it's incompatible with the stimulus, but the stimulus doesn't say it and wouldn't be improved by having it.

The stimulus doesn't say or require anything about improved computer models making cars safer. Instead, it seems like improved computer models could allow cars to be produced as safely, but for less money, because less money would have to be spent on safety for the same result. So the stimulus seems to predict the same level of safety for less cost. More safety is thus out of scope.

Robert Carroll
User avatar
 ashpine17
  • Posts: 321
  • Joined: Apr 06, 2021
|
#91642
I don't understan why B is problematic. I get that it reaffirms the first part of the conditional statement but why is that an issue?
 Robert Carroll
PowerScore Staff
  • PowerScore Staff
  • Posts: 1787
  • Joined: Dec 06, 2013
|
#92900
ashpine,

Because providing the sufficient condition is useless for proving a conditional.

An example:

Conclusion: "If you travel by car, you're more likely to get into an accident."

Supposed premise: "You are going to travel by car."

This is exactly analogous - the stimulus contains a conditional as its conclusion and answer choice (B) is affirming the sufficient condition of the conditional. But...how does proving that you're traveling by car prove that you're more likely to get into an accident?

If this inference form worked, we could prove anything with anything:

Premise: A

Conclusion: A :arrow: B

We could then use conditional logic on that in a subsequent argument to prove:

B

So from any A, we could prove any B.

Robert Carroll

Get the most out of your LSAT Prep Plus subscription.

Analyze and track your performance with our Testing and Analytics Package.